Give an example of: A group with an element A of order 3, an element B with order 4, where order of AB is...











up vote
18
down vote

favorite
4












I'm a mathematics major studying at University as an undergrad. This is a question on the study guide for the upcoming final in Math 344 - Group Theory:



"Give an example of a group G with an element a of order 3, an element b of order 4, where order of ab is less than 12."



My understanding is if an element a has order n, it means that if a is combined with itself n times, it results in the identity element e: a^n=e. It also means that there is no number smaller than n where this is true for a. Two elements a and b can be combined into ab such that ab is the result of whatever operator acts on the group. Example: If the operator is addition, ab=a+b



Possible groups I've considered that don't seem to work:



-D2n, the group of symmetries of a regular n-sided polygon. This includes rotations about the center, or flips across lines that go through the center. It doesn't seem to work because if a rotation has order 3, and another rotation has order 4, their combination should have order 12. All the flips or combinations of a rotation with a flip have order 2



-Quotient group Z/nZ. Z/12Z doesn't seem to work, since {12Z+4} is order 3, {12Z+3} is order 4, but {12Z+3+12Z+4}={12Z+7}, which has order 12. This seems to hold for other values of n



-The group of integers/reals/rationals with the addition operator, or the group of non-zero real numbers with multiplication, or the group of rationals with multiplication. None of these seem to have elements of order 3 or 4 in the first place



These are the main groups we worked with in class. I've searched this site and others for examples of groups I may have overlooked, with no luck. I believe the elements I need won't be commutative - such that ab does not equal ba - but I'm not certain.



Thank you!










share|cite|improve this question


























    up vote
    18
    down vote

    favorite
    4












    I'm a mathematics major studying at University as an undergrad. This is a question on the study guide for the upcoming final in Math 344 - Group Theory:



    "Give an example of a group G with an element a of order 3, an element b of order 4, where order of ab is less than 12."



    My understanding is if an element a has order n, it means that if a is combined with itself n times, it results in the identity element e: a^n=e. It also means that there is no number smaller than n where this is true for a. Two elements a and b can be combined into ab such that ab is the result of whatever operator acts on the group. Example: If the operator is addition, ab=a+b



    Possible groups I've considered that don't seem to work:



    -D2n, the group of symmetries of a regular n-sided polygon. This includes rotations about the center, or flips across lines that go through the center. It doesn't seem to work because if a rotation has order 3, and another rotation has order 4, their combination should have order 12. All the flips or combinations of a rotation with a flip have order 2



    -Quotient group Z/nZ. Z/12Z doesn't seem to work, since {12Z+4} is order 3, {12Z+3} is order 4, but {12Z+3+12Z+4}={12Z+7}, which has order 12. This seems to hold for other values of n



    -The group of integers/reals/rationals with the addition operator, or the group of non-zero real numbers with multiplication, or the group of rationals with multiplication. None of these seem to have elements of order 3 or 4 in the first place



    These are the main groups we worked with in class. I've searched this site and others for examples of groups I may have overlooked, with no luck. I believe the elements I need won't be commutative - such that ab does not equal ba - but I'm not certain.



    Thank you!










    share|cite|improve this question
























      up vote
      18
      down vote

      favorite
      4









      up vote
      18
      down vote

      favorite
      4






      4





      I'm a mathematics major studying at University as an undergrad. This is a question on the study guide for the upcoming final in Math 344 - Group Theory:



      "Give an example of a group G with an element a of order 3, an element b of order 4, where order of ab is less than 12."



      My understanding is if an element a has order n, it means that if a is combined with itself n times, it results in the identity element e: a^n=e. It also means that there is no number smaller than n where this is true for a. Two elements a and b can be combined into ab such that ab is the result of whatever operator acts on the group. Example: If the operator is addition, ab=a+b



      Possible groups I've considered that don't seem to work:



      -D2n, the group of symmetries of a regular n-sided polygon. This includes rotations about the center, or flips across lines that go through the center. It doesn't seem to work because if a rotation has order 3, and another rotation has order 4, their combination should have order 12. All the flips or combinations of a rotation with a flip have order 2



      -Quotient group Z/nZ. Z/12Z doesn't seem to work, since {12Z+4} is order 3, {12Z+3} is order 4, but {12Z+3+12Z+4}={12Z+7}, which has order 12. This seems to hold for other values of n



      -The group of integers/reals/rationals with the addition operator, or the group of non-zero real numbers with multiplication, or the group of rationals with multiplication. None of these seem to have elements of order 3 or 4 in the first place



      These are the main groups we worked with in class. I've searched this site and others for examples of groups I may have overlooked, with no luck. I believe the elements I need won't be commutative - such that ab does not equal ba - but I'm not certain.



      Thank you!










      share|cite|improve this question













      I'm a mathematics major studying at University as an undergrad. This is a question on the study guide for the upcoming final in Math 344 - Group Theory:



      "Give an example of a group G with an element a of order 3, an element b of order 4, where order of ab is less than 12."



      My understanding is if an element a has order n, it means that if a is combined with itself n times, it results in the identity element e: a^n=e. It also means that there is no number smaller than n where this is true for a. Two elements a and b can be combined into ab such that ab is the result of whatever operator acts on the group. Example: If the operator is addition, ab=a+b



      Possible groups I've considered that don't seem to work:



      -D2n, the group of symmetries of a regular n-sided polygon. This includes rotations about the center, or flips across lines that go through the center. It doesn't seem to work because if a rotation has order 3, and another rotation has order 4, their combination should have order 12. All the flips or combinations of a rotation with a flip have order 2



      -Quotient group Z/nZ. Z/12Z doesn't seem to work, since {12Z+4} is order 3, {12Z+3} is order 4, but {12Z+3+12Z+4}={12Z+7}, which has order 12. This seems to hold for other values of n



      -The group of integers/reals/rationals with the addition operator, or the group of non-zero real numbers with multiplication, or the group of rationals with multiplication. None of these seem to have elements of order 3 or 4 in the first place



      These are the main groups we worked with in class. I've searched this site and others for examples of groups I may have overlooked, with no luck. I believe the elements I need won't be commutative - such that ab does not equal ba - but I'm not certain.



      Thank you!







      group-theory quotient-group






      share|cite|improve this question













      share|cite|improve this question











      share|cite|improve this question




      share|cite|improve this question










      asked Dec 5 at 18:48









      Jay Ess

      944




      944






















          4 Answers
          4






          active

          oldest

          votes

















          up vote
          32
          down vote













          You are right to have observed that your elements can't commute.



          The symmetric group $S_4$ has elements of both kinds but no element of order $12$ so you have lots of choices.



          I hope the symmetric groups soon become a go to place for examples.






          share|cite|improve this answer






























            up vote
            6
            down vote













            Let $A$ and $B$ be the quaternions $i$ and $cos(2pi/3) + jsin(2pi/3)$. Then $AB = icos(2pi/3)+ksin(2pi/3)$. Since $AB$ is a vector quaternion of unit length, it follows that $(AB)^2=-1$. So $AB$ has order $4$.



            The resulting group is $mbox{Dic}_3$. See dicyclic groups.






            share|cite|improve this answer






























              up vote
              4
              down vote













              Consider
              begin{align*}
              a = pmatrix{1 & 1 \ 0 & 1}, ;;b = pmatrix{0 & -1 \ 1 & 0}
              end{align*}

              in $SL_2(mathbb{F}_3)$. The product
              begin{align*}
              ab &= pmatrix{1 & -1 \ 1 & 0}
              end{align*}

              has $(ab)^3 = -1$.






              share|cite|improve this answer





















              • You might want to mention explicitly that this means $ab$ has order $3$ (and, for ease of understanding, change $-1$ to $-I$ or better yet write out the matrix).
                – Greg Martin
                Dec 6 at 8:26






              • 1




                I thought it meant that ab has order 6 (because order is the smallest exponent to get to 1)
                – Pacopaco
                Dec 6 at 15:39




















              up vote
              3
              down vote













              Note: I misread the question as just asking for it to not be equal to 12. To be less than 12, you could probably proceed with the same strategy, but it would be a bit more complicated.



              For order(ab) greater than 12, let $G_1$ be the free group of $(a,b)$, $H_1$ be the free group of $(a^3,b^4)$, $G_2$ be the quotient group of $G_1/H_1$. $a$ is order $3$ in $G_2$, $b$ is order $4$, and $ab$ has infinite order.



              Put in slightly less technical terms, $G_1$ is the group formed from all possible sequences of $a$, $b$, $a^{-1}$ and $b^{-1}$. E.g. $a^3ba^{-4}$ would be one element. $G_2$ is this group, except that, given two sequences, if we can go from one to the other by inserting $a^3$, $a^{-3}$, $b^4$, and/or $b^{-4}$, then we consider the two sequences to be the same. In other words, we create $G_2$ by taking $G_1$ and simply defining $a^3$ and $b^4$ to be the identity.



              One physical interpretation would be suppose we have two disks. The first can only be turned in increments of 120 degrees (clockwise or counterclockwise), and the second only in increments of 90 degrees. Let $G_2$ be the set of sequences of rotations of the two disks, where a sequence is considered different if it does the disks in different order (rotating the first disk, then the second, is different from rotating the second then the first), but the same if, while we're on a particular disk, it ends up at the same place. So rotating the first disk four time is the same as rotating the first disk once.






              share|cite|improve this answer























              • +1 very nice, though probably not accessible yet for the OP.
                – Ethan Bolker
                Dec 5 at 23:39










              • A nice example, but the order of $ab$ is supposed to be less than 12.
                – Alex S
                Dec 6 at 0:43










              • Your $H_1$ is not normal in $G_1$ (finitely generated subgroup of infinite index is never normal in noncommutative free group). After taking normal closure your construction is fine, but writing blatantly wrong statements is not fine (I think).
                – xsnl
                Dec 6 at 13:37













              Your Answer





              StackExchange.ifUsing("editor", function () {
              return StackExchange.using("mathjaxEditing", function () {
              StackExchange.MarkdownEditor.creationCallbacks.add(function (editor, postfix) {
              StackExchange.mathjaxEditing.prepareWmdForMathJax(editor, postfix, [["$", "$"], ["\\(","\\)"]]);
              });
              });
              }, "mathjax-editing");

              StackExchange.ready(function() {
              var channelOptions = {
              tags: "".split(" "),
              id: "69"
              };
              initTagRenderer("".split(" "), "".split(" "), channelOptions);

              StackExchange.using("externalEditor", function() {
              // Have to fire editor after snippets, if snippets enabled
              if (StackExchange.settings.snippets.snippetsEnabled) {
              StackExchange.using("snippets", function() {
              createEditor();
              });
              }
              else {
              createEditor();
              }
              });

              function createEditor() {
              StackExchange.prepareEditor({
              heartbeatType: 'answer',
              convertImagesToLinks: true,
              noModals: true,
              showLowRepImageUploadWarning: true,
              reputationToPostImages: 10,
              bindNavPrevention: true,
              postfix: "",
              imageUploader: {
              brandingHtml: "Powered by u003ca class="icon-imgur-white" href="https://imgur.com/"u003eu003c/au003e",
              contentPolicyHtml: "User contributions licensed under u003ca href="https://creativecommons.org/licenses/by-sa/3.0/"u003ecc by-sa 3.0 with attribution requiredu003c/au003e u003ca href="https://stackoverflow.com/legal/content-policy"u003e(content policy)u003c/au003e",
              allowUrls: true
              },
              noCode: true, onDemand: true,
              discardSelector: ".discard-answer"
              ,immediatelyShowMarkdownHelp:true
              });


              }
              });














              draft saved

              draft discarded


















              StackExchange.ready(
              function () {
              StackExchange.openid.initPostLogin('.new-post-login', 'https%3a%2f%2fmath.stackexchange.com%2fquestions%2f3027482%2fgive-an-example-of-a-group-with-an-element-a-of-order-3-an-element-b-with-orde%23new-answer', 'question_page');
              }
              );

              Post as a guest















              Required, but never shown

























              4 Answers
              4






              active

              oldest

              votes








              4 Answers
              4






              active

              oldest

              votes









              active

              oldest

              votes






              active

              oldest

              votes








              up vote
              32
              down vote













              You are right to have observed that your elements can't commute.



              The symmetric group $S_4$ has elements of both kinds but no element of order $12$ so you have lots of choices.



              I hope the symmetric groups soon become a go to place for examples.






              share|cite|improve this answer



























                up vote
                32
                down vote













                You are right to have observed that your elements can't commute.



                The symmetric group $S_4$ has elements of both kinds but no element of order $12$ so you have lots of choices.



                I hope the symmetric groups soon become a go to place for examples.






                share|cite|improve this answer

























                  up vote
                  32
                  down vote










                  up vote
                  32
                  down vote









                  You are right to have observed that your elements can't commute.



                  The symmetric group $S_4$ has elements of both kinds but no element of order $12$ so you have lots of choices.



                  I hope the symmetric groups soon become a go to place for examples.






                  share|cite|improve this answer














                  You are right to have observed that your elements can't commute.



                  The symmetric group $S_4$ has elements of both kinds but no element of order $12$ so you have lots of choices.



                  I hope the symmetric groups soon become a go to place for examples.







                  share|cite|improve this answer














                  share|cite|improve this answer



                  share|cite|improve this answer








                  edited Dec 5 at 19:07

























                  answered Dec 5 at 18:50









                  Ethan Bolker

                  40.7k546108




                  40.7k546108






















                      up vote
                      6
                      down vote













                      Let $A$ and $B$ be the quaternions $i$ and $cos(2pi/3) + jsin(2pi/3)$. Then $AB = icos(2pi/3)+ksin(2pi/3)$. Since $AB$ is a vector quaternion of unit length, it follows that $(AB)^2=-1$. So $AB$ has order $4$.



                      The resulting group is $mbox{Dic}_3$. See dicyclic groups.






                      share|cite|improve this answer



























                        up vote
                        6
                        down vote













                        Let $A$ and $B$ be the quaternions $i$ and $cos(2pi/3) + jsin(2pi/3)$. Then $AB = icos(2pi/3)+ksin(2pi/3)$. Since $AB$ is a vector quaternion of unit length, it follows that $(AB)^2=-1$. So $AB$ has order $4$.



                        The resulting group is $mbox{Dic}_3$. See dicyclic groups.






                        share|cite|improve this answer

























                          up vote
                          6
                          down vote










                          up vote
                          6
                          down vote









                          Let $A$ and $B$ be the quaternions $i$ and $cos(2pi/3) + jsin(2pi/3)$. Then $AB = icos(2pi/3)+ksin(2pi/3)$. Since $AB$ is a vector quaternion of unit length, it follows that $(AB)^2=-1$. So $AB$ has order $4$.



                          The resulting group is $mbox{Dic}_3$. See dicyclic groups.






                          share|cite|improve this answer














                          Let $A$ and $B$ be the quaternions $i$ and $cos(2pi/3) + jsin(2pi/3)$. Then $AB = icos(2pi/3)+ksin(2pi/3)$. Since $AB$ is a vector quaternion of unit length, it follows that $(AB)^2=-1$. So $AB$ has order $4$.



                          The resulting group is $mbox{Dic}_3$. See dicyclic groups.







                          share|cite|improve this answer














                          share|cite|improve this answer



                          share|cite|improve this answer








                          edited Dec 6 at 17:49

























                          answered Dec 6 at 10:33









                          man on laptop

                          5,69611238




                          5,69611238






















                              up vote
                              4
                              down vote













                              Consider
                              begin{align*}
                              a = pmatrix{1 & 1 \ 0 & 1}, ;;b = pmatrix{0 & -1 \ 1 & 0}
                              end{align*}

                              in $SL_2(mathbb{F}_3)$. The product
                              begin{align*}
                              ab &= pmatrix{1 & -1 \ 1 & 0}
                              end{align*}

                              has $(ab)^3 = -1$.






                              share|cite|improve this answer





















                              • You might want to mention explicitly that this means $ab$ has order $3$ (and, for ease of understanding, change $-1$ to $-I$ or better yet write out the matrix).
                                – Greg Martin
                                Dec 6 at 8:26






                              • 1




                                I thought it meant that ab has order 6 (because order is the smallest exponent to get to 1)
                                – Pacopaco
                                Dec 6 at 15:39

















                              up vote
                              4
                              down vote













                              Consider
                              begin{align*}
                              a = pmatrix{1 & 1 \ 0 & 1}, ;;b = pmatrix{0 & -1 \ 1 & 0}
                              end{align*}

                              in $SL_2(mathbb{F}_3)$. The product
                              begin{align*}
                              ab &= pmatrix{1 & -1 \ 1 & 0}
                              end{align*}

                              has $(ab)^3 = -1$.






                              share|cite|improve this answer





















                              • You might want to mention explicitly that this means $ab$ has order $3$ (and, for ease of understanding, change $-1$ to $-I$ or better yet write out the matrix).
                                – Greg Martin
                                Dec 6 at 8:26






                              • 1




                                I thought it meant that ab has order 6 (because order is the smallest exponent to get to 1)
                                – Pacopaco
                                Dec 6 at 15:39















                              up vote
                              4
                              down vote










                              up vote
                              4
                              down vote









                              Consider
                              begin{align*}
                              a = pmatrix{1 & 1 \ 0 & 1}, ;;b = pmatrix{0 & -1 \ 1 & 0}
                              end{align*}

                              in $SL_2(mathbb{F}_3)$. The product
                              begin{align*}
                              ab &= pmatrix{1 & -1 \ 1 & 0}
                              end{align*}

                              has $(ab)^3 = -1$.






                              share|cite|improve this answer












                              Consider
                              begin{align*}
                              a = pmatrix{1 & 1 \ 0 & 1}, ;;b = pmatrix{0 & -1 \ 1 & 0}
                              end{align*}

                              in $SL_2(mathbb{F}_3)$. The product
                              begin{align*}
                              ab &= pmatrix{1 & -1 \ 1 & 0}
                              end{align*}

                              has $(ab)^3 = -1$.







                              share|cite|improve this answer












                              share|cite|improve this answer



                              share|cite|improve this answer










                              answered Dec 6 at 5:56









                              anomaly

                              17.3k42662




                              17.3k42662












                              • You might want to mention explicitly that this means $ab$ has order $3$ (and, for ease of understanding, change $-1$ to $-I$ or better yet write out the matrix).
                                – Greg Martin
                                Dec 6 at 8:26






                              • 1




                                I thought it meant that ab has order 6 (because order is the smallest exponent to get to 1)
                                – Pacopaco
                                Dec 6 at 15:39




















                              • You might want to mention explicitly that this means $ab$ has order $3$ (and, for ease of understanding, change $-1$ to $-I$ or better yet write out the matrix).
                                – Greg Martin
                                Dec 6 at 8:26






                              • 1




                                I thought it meant that ab has order 6 (because order is the smallest exponent to get to 1)
                                – Pacopaco
                                Dec 6 at 15:39


















                              You might want to mention explicitly that this means $ab$ has order $3$ (and, for ease of understanding, change $-1$ to $-I$ or better yet write out the matrix).
                              – Greg Martin
                              Dec 6 at 8:26




                              You might want to mention explicitly that this means $ab$ has order $3$ (and, for ease of understanding, change $-1$ to $-I$ or better yet write out the matrix).
                              – Greg Martin
                              Dec 6 at 8:26




                              1




                              1




                              I thought it meant that ab has order 6 (because order is the smallest exponent to get to 1)
                              – Pacopaco
                              Dec 6 at 15:39






                              I thought it meant that ab has order 6 (because order is the smallest exponent to get to 1)
                              – Pacopaco
                              Dec 6 at 15:39












                              up vote
                              3
                              down vote













                              Note: I misread the question as just asking for it to not be equal to 12. To be less than 12, you could probably proceed with the same strategy, but it would be a bit more complicated.



                              For order(ab) greater than 12, let $G_1$ be the free group of $(a,b)$, $H_1$ be the free group of $(a^3,b^4)$, $G_2$ be the quotient group of $G_1/H_1$. $a$ is order $3$ in $G_2$, $b$ is order $4$, and $ab$ has infinite order.



                              Put in slightly less technical terms, $G_1$ is the group formed from all possible sequences of $a$, $b$, $a^{-1}$ and $b^{-1}$. E.g. $a^3ba^{-4}$ would be one element. $G_2$ is this group, except that, given two sequences, if we can go from one to the other by inserting $a^3$, $a^{-3}$, $b^4$, and/or $b^{-4}$, then we consider the two sequences to be the same. In other words, we create $G_2$ by taking $G_1$ and simply defining $a^3$ and $b^4$ to be the identity.



                              One physical interpretation would be suppose we have two disks. The first can only be turned in increments of 120 degrees (clockwise or counterclockwise), and the second only in increments of 90 degrees. Let $G_2$ be the set of sequences of rotations of the two disks, where a sequence is considered different if it does the disks in different order (rotating the first disk, then the second, is different from rotating the second then the first), but the same if, while we're on a particular disk, it ends up at the same place. So rotating the first disk four time is the same as rotating the first disk once.






                              share|cite|improve this answer























                              • +1 very nice, though probably not accessible yet for the OP.
                                – Ethan Bolker
                                Dec 5 at 23:39










                              • A nice example, but the order of $ab$ is supposed to be less than 12.
                                – Alex S
                                Dec 6 at 0:43










                              • Your $H_1$ is not normal in $G_1$ (finitely generated subgroup of infinite index is never normal in noncommutative free group). After taking normal closure your construction is fine, but writing blatantly wrong statements is not fine (I think).
                                – xsnl
                                Dec 6 at 13:37

















                              up vote
                              3
                              down vote













                              Note: I misread the question as just asking for it to not be equal to 12. To be less than 12, you could probably proceed with the same strategy, but it would be a bit more complicated.



                              For order(ab) greater than 12, let $G_1$ be the free group of $(a,b)$, $H_1$ be the free group of $(a^3,b^4)$, $G_2$ be the quotient group of $G_1/H_1$. $a$ is order $3$ in $G_2$, $b$ is order $4$, and $ab$ has infinite order.



                              Put in slightly less technical terms, $G_1$ is the group formed from all possible sequences of $a$, $b$, $a^{-1}$ and $b^{-1}$. E.g. $a^3ba^{-4}$ would be one element. $G_2$ is this group, except that, given two sequences, if we can go from one to the other by inserting $a^3$, $a^{-3}$, $b^4$, and/or $b^{-4}$, then we consider the two sequences to be the same. In other words, we create $G_2$ by taking $G_1$ and simply defining $a^3$ and $b^4$ to be the identity.



                              One physical interpretation would be suppose we have two disks. The first can only be turned in increments of 120 degrees (clockwise or counterclockwise), and the second only in increments of 90 degrees. Let $G_2$ be the set of sequences of rotations of the two disks, where a sequence is considered different if it does the disks in different order (rotating the first disk, then the second, is different from rotating the second then the first), but the same if, while we're on a particular disk, it ends up at the same place. So rotating the first disk four time is the same as rotating the first disk once.






                              share|cite|improve this answer























                              • +1 very nice, though probably not accessible yet for the OP.
                                – Ethan Bolker
                                Dec 5 at 23:39










                              • A nice example, but the order of $ab$ is supposed to be less than 12.
                                – Alex S
                                Dec 6 at 0:43










                              • Your $H_1$ is not normal in $G_1$ (finitely generated subgroup of infinite index is never normal in noncommutative free group). After taking normal closure your construction is fine, but writing blatantly wrong statements is not fine (I think).
                                – xsnl
                                Dec 6 at 13:37















                              up vote
                              3
                              down vote










                              up vote
                              3
                              down vote









                              Note: I misread the question as just asking for it to not be equal to 12. To be less than 12, you could probably proceed with the same strategy, but it would be a bit more complicated.



                              For order(ab) greater than 12, let $G_1$ be the free group of $(a,b)$, $H_1$ be the free group of $(a^3,b^4)$, $G_2$ be the quotient group of $G_1/H_1$. $a$ is order $3$ in $G_2$, $b$ is order $4$, and $ab$ has infinite order.



                              Put in slightly less technical terms, $G_1$ is the group formed from all possible sequences of $a$, $b$, $a^{-1}$ and $b^{-1}$. E.g. $a^3ba^{-4}$ would be one element. $G_2$ is this group, except that, given two sequences, if we can go from one to the other by inserting $a^3$, $a^{-3}$, $b^4$, and/or $b^{-4}$, then we consider the two sequences to be the same. In other words, we create $G_2$ by taking $G_1$ and simply defining $a^3$ and $b^4$ to be the identity.



                              One physical interpretation would be suppose we have two disks. The first can only be turned in increments of 120 degrees (clockwise or counterclockwise), and the second only in increments of 90 degrees. Let $G_2$ be the set of sequences of rotations of the two disks, where a sequence is considered different if it does the disks in different order (rotating the first disk, then the second, is different from rotating the second then the first), but the same if, while we're on a particular disk, it ends up at the same place. So rotating the first disk four time is the same as rotating the first disk once.






                              share|cite|improve this answer














                              Note: I misread the question as just asking for it to not be equal to 12. To be less than 12, you could probably proceed with the same strategy, but it would be a bit more complicated.



                              For order(ab) greater than 12, let $G_1$ be the free group of $(a,b)$, $H_1$ be the free group of $(a^3,b^4)$, $G_2$ be the quotient group of $G_1/H_1$. $a$ is order $3$ in $G_2$, $b$ is order $4$, and $ab$ has infinite order.



                              Put in slightly less technical terms, $G_1$ is the group formed from all possible sequences of $a$, $b$, $a^{-1}$ and $b^{-1}$. E.g. $a^3ba^{-4}$ would be one element. $G_2$ is this group, except that, given two sequences, if we can go from one to the other by inserting $a^3$, $a^{-3}$, $b^4$, and/or $b^{-4}$, then we consider the two sequences to be the same. In other words, we create $G_2$ by taking $G_1$ and simply defining $a^3$ and $b^4$ to be the identity.



                              One physical interpretation would be suppose we have two disks. The first can only be turned in increments of 120 degrees (clockwise or counterclockwise), and the second only in increments of 90 degrees. Let $G_2$ be the set of sequences of rotations of the two disks, where a sequence is considered different if it does the disks in different order (rotating the first disk, then the second, is different from rotating the second then the first), but the same if, while we're on a particular disk, it ends up at the same place. So rotating the first disk four time is the same as rotating the first disk once.







                              share|cite|improve this answer














                              share|cite|improve this answer



                              share|cite|improve this answer








                              edited Dec 6 at 1:00

























                              answered Dec 5 at 23:33









                              Acccumulation

                              6,6512616




                              6,6512616












                              • +1 very nice, though probably not accessible yet for the OP.
                                – Ethan Bolker
                                Dec 5 at 23:39










                              • A nice example, but the order of $ab$ is supposed to be less than 12.
                                – Alex S
                                Dec 6 at 0:43










                              • Your $H_1$ is not normal in $G_1$ (finitely generated subgroup of infinite index is never normal in noncommutative free group). After taking normal closure your construction is fine, but writing blatantly wrong statements is not fine (I think).
                                – xsnl
                                Dec 6 at 13:37




















                              • +1 very nice, though probably not accessible yet for the OP.
                                – Ethan Bolker
                                Dec 5 at 23:39










                              • A nice example, but the order of $ab$ is supposed to be less than 12.
                                – Alex S
                                Dec 6 at 0:43










                              • Your $H_1$ is not normal in $G_1$ (finitely generated subgroup of infinite index is never normal in noncommutative free group). After taking normal closure your construction is fine, but writing blatantly wrong statements is not fine (I think).
                                – xsnl
                                Dec 6 at 13:37


















                              +1 very nice, though probably not accessible yet for the OP.
                              – Ethan Bolker
                              Dec 5 at 23:39




                              +1 very nice, though probably not accessible yet for the OP.
                              – Ethan Bolker
                              Dec 5 at 23:39












                              A nice example, but the order of $ab$ is supposed to be less than 12.
                              – Alex S
                              Dec 6 at 0:43




                              A nice example, but the order of $ab$ is supposed to be less than 12.
                              – Alex S
                              Dec 6 at 0:43












                              Your $H_1$ is not normal in $G_1$ (finitely generated subgroup of infinite index is never normal in noncommutative free group). After taking normal closure your construction is fine, but writing blatantly wrong statements is not fine (I think).
                              – xsnl
                              Dec 6 at 13:37






                              Your $H_1$ is not normal in $G_1$ (finitely generated subgroup of infinite index is never normal in noncommutative free group). After taking normal closure your construction is fine, but writing blatantly wrong statements is not fine (I think).
                              – xsnl
                              Dec 6 at 13:37




















                              draft saved

                              draft discarded




















































                              Thanks for contributing an answer to Mathematics Stack Exchange!


                              • Please be sure to answer the question. Provide details and share your research!

                              But avoid



                              • Asking for help, clarification, or responding to other answers.

                              • Making statements based on opinion; back them up with references or personal experience.


                              Use MathJax to format equations. MathJax reference.


                              To learn more, see our tips on writing great answers.





                              Some of your past answers have not been well-received, and you're in danger of being blocked from answering.


                              Please pay close attention to the following guidance:


                              • Please be sure to answer the question. Provide details and share your research!

                              But avoid



                              • Asking for help, clarification, or responding to other answers.

                              • Making statements based on opinion; back them up with references or personal experience.


                              To learn more, see our tips on writing great answers.




                              draft saved


                              draft discarded














                              StackExchange.ready(
                              function () {
                              StackExchange.openid.initPostLogin('.new-post-login', 'https%3a%2f%2fmath.stackexchange.com%2fquestions%2f3027482%2fgive-an-example-of-a-group-with-an-element-a-of-order-3-an-element-b-with-orde%23new-answer', 'question_page');
                              }
                              );

                              Post as a guest















                              Required, but never shown





















































                              Required, but never shown














                              Required, but never shown












                              Required, but never shown







                              Required, but never shown

































                              Required, but never shown














                              Required, but never shown












                              Required, but never shown







                              Required, but never shown







                              Popular posts from this blog

                              flock() on closed filehandle LOCK_FILE at /usr/bin/apt-mirror

                              Mangá

                               ⁒  ․,‪⁊‑⁙ ⁖, ⁇‒※‌, †,⁖‗‌⁝    ‾‸⁘,‖⁔⁣,⁂‾
”‑,‥–,‬ ,⁀‹⁋‴⁑ ‒ ,‴⁋”‼ ⁨,‷⁔„ ‰′,‐‚ ‥‡‎“‷⁃⁨⁅⁣,⁔
⁇‘⁔⁡⁏⁌⁡‿‶‏⁨ ⁣⁕⁖⁨⁩⁥‽⁀  ‴‬⁜‟ ⁃‣‧⁕‮ …‍⁨‴ ⁩,⁚⁖‫ ,‵ ⁀,‮⁝‣‣ ⁑  ⁂– ․, ‾‽ ‏⁁“⁗‸ ‾… ‹‡⁌⁎‸‘ ‡⁏⁌‪ ‵⁛ ‎⁨ ―⁦⁤⁄⁕